Last visit was: 26 Apr 2024, 05:26 It is currently 26 Apr 2024, 05:26

Close
GMAT Club Daily Prep
Thank you for using the timer - this advanced tool can estimate your performance and suggest more practice questions. We have subscribed you to Daily Prep Questions via email.

Customized
for You

we will pick new questions that match your level based on your Timer History

Track
Your Progress

every week, we’ll send you an estimated GMAT score based on your performance

Practice
Pays

we will pick new questions that match your level based on your Timer History
Not interested in getting valuable practice questions and articles delivered to your email? No problem, unsubscribe here.
Close
Request Expert Reply
Confirm Cancel
SORT BY:
Date
Math Expert
Joined: 02 Sep 2009
Posts: 92931
Own Kudos [?]: 619154 [2]
Given Kudos: 81609
Send PM
Senior Manager
Senior Manager
Joined: 13 Oct 2016
Posts: 300
Own Kudos [?]: 768 [3]
Given Kudos: 40
GPA: 3.98
Send PM
Intern
Intern
Joined: 10 Jun 2016
Posts: 32
Own Kudos [?]: 12 [0]
Given Kudos: 194
Schools: IIM-A"19
Send PM
Math Revolution GMAT Instructor
Joined: 16 Aug 2015
Posts: 10161
Own Kudos [?]: 16598 [0]
Given Kudos: 4
GMAT 1: 760 Q51 V42
GPA: 3.82
Send PM
Re: Is m divisible by n? [#permalink]
Expert Reply
Bunuel wrote:
Is m divisible by n?

(1) mn/2 is divisible by n
(2) m + n is divisible by n.


Forget conventional ways of solving math questions. For DS problems, the VA (Variable Approach) method is the quickest and easiest way to find the answer without actually solving the problem. Remember that equal numbers of variables and independent equations ensure a solution.

Since we have 2 variables (m and n) and 0 equations, C is most likely to be the answer. So, we should consider conditions 1) & 2) together first. After comparing the number of variables and the number of equations, we can save time by considering conditions 1) & 2) together first.

Conditions 1) & 2)
m + n = nk from the condition 2).
m = (k-1)n
Thus, both conditions together are not sufficient.

Since this question is an integer question (one of the key question areas), CMT (Common Mistake Type) 4(A) of the VA

(Variable Approach) method tells us that we should also check answers A and B.

Condition 1)
If m = 2, n = 2, the answer is 'yes'.
If m = 2, n = 3, the answer is 'no'.
Thus the condition 1) is not sufficient.

Condition 2)
m + n = nk from the condition 2).
m = (k-1)n
Thus, the condition 2) is sufficient.

Therefore, the answer is B.

Normally, in problems which require 2 equations, such as those in which the original conditions include 2 variables, or 3 variables and 1 equation, or 4 variables and 2 equations, each of conditions 1) and 2) provide an additional equation. In these problems, the two key possibilities are that C is the answer (with probability 70%), and E is the answer (with probability 25%). Thus, there is only a 5% chance that A, B or D is the answer. This occurs in common mistake types 3 and 4. Since C (both conditions together are sufficient) is the most likely answer, we save time by first checking whether conditions 1) and 2) are sufficient, when taken together. Obviously, there may be cases in which the answer is A, B, D or E, but if conditions 1) and 2) are NOT sufficient when taken together, the answer must be E.
e-GMAT Representative
Joined: 04 Jan 2015
Posts: 3726
Own Kudos [?]: 16844 [0]
Given Kudos: 165
Send PM
Re: Is m divisible by n? [#permalink]
Expert Reply

Solution:



To find:

    • If m is divisible by n or not.

      o Inference: if m is divisible by n, m should be a multiple of n.

         This means, m can be represented in the form of: nx, where x is any positive integer.

Working out:

Statement 1:

mn/2 is divisible by n

This means, we can write mn/2 as a multiple of n

Translating the above statement in the mathematical form, we get:

\(mn/ 2 = nK\), where K is a positive integer.

Dividing both the sides of this equation by n, we get:

\(m/2 = K\)

or, \(m =2K\).

Since m is not of the form of nx, Statement 1 alone is not sufficient to answer this question.

Statement 2:

m + n is divisible by n

    • From this statement, we can write that m+n = nK, where K is any positive integer.
Dividing both the sides of this equation by n, we get:

    •\(m/n + 1 = K\)

Since K is an integer, and 1 is also an integer, m/n has to be an integer.

    • Or, m/n can be written as y, where y is any integer.

    • Thus, m/n = y, Or m = y*n

Since m can be represented in the form of n*a positive integer, m is divisible by n.

Hence, Statement 2 alone is sufficient to answer our question.

Answer: Option B
GMAT Club Bot
Re: Is m divisible by n? [#permalink]
Moderator:
Math Expert
92929 posts

Powered by phpBB © phpBB Group | Emoji artwork provided by EmojiOne